[obm-l] Re: [obm-l] ARCOS NOTÁVEIS

2003-07-15 Por tôpico Nicolau C. Saldanha
On Sat, Jul 12, 2003 at 12:16:09AM -0300, Nelson alotiab wrote:
> Olá, é a primeira vez que escrevo para a lista. Inicialmente, gostaria de
> parabenizar os idealizadores dessa lista, pois, por incrível que pareça,
> sites (de qualidade) educacionais e voltados ao ensino, são escassos na
> internet, especialmente, os de lingua portuguesa.

Obrigado pela parte que me toca. Claro que a boa qualidade da lista 'e
devida aos que nela escrevem material de qualidade.

> Pois bem... agora vai
> minha dúvida: Gostaria de uma informação mais esclarecedora sobre arcos
> notáveis. Não entendi a "relação" (não sei se esse termo é apropriado)
> sen("pi"/n) = Ln/2, n E N e n >= 3 (obs: Ln = lado do poligono regular de n
> lados inscrito no ciclo). Sei que minha dúvida é simples, e até boba, mas, se
> possível, gostaria de uma explicação mais detalhada. 

Talvez uma solu,c~ao por anal'itica deixe voc^e satisfeito?
Os v'ertices de um pol'igono regular de n lados s~ao da forma
(cos(a+(2*pi*k/n)), sen(a+(2*pi*k/n))) onde k est um inteiro entre 0 e n-1.
Voc^e pode tomar a = -pi/n e ent~ao ter'a os v'ertices
(cos(pi/n), sen(pi/n)), (cos(pi/n), -sen(pi/n)):
a dist^encia entre estes dois v'ertices 'e igual a Ln = 2*sen(pi/n).

[]s, N.
=
Instruções para entrar na lista, sair da lista e usar a lista em
http://www.mat.puc-rio.br/~nicolau/olimp/obm-l.html
=


Re: [obm-l] Referencia Bibliografica (era: Combinatoria (In off))

2003-07-15 Por tôpico Nicolau C. Saldanha
On Tue, Jul 15, 2003 at 05:02:02PM -0300, Manuel Valentim Pera wrote:
> Paulo,
> 
>   Boa tarde,
> 
> On Tue, 15 Jul 2003, Paulo Santa Rita wrote:
> 
> > 
> > Voce nao gostaria de apresentar aqui uma construcao dos reais, via cortes  
> > ou sequencias de Cauchy, por exemplo, e desta construcao derivar o TEOREMA 
> > DO SUPREMO ?
> > 
> 
>  Nao teria sentido fazer isso aqui, mas recomendo a leitura dos excelentes
> textos abaixo em que isso esta' bem feito, muito mais bem feito do que
> qualquer coisa que eu pudesse esbocar aqui (eles podem ser encontrados em
> bibliotecas de faculdades de Matematica):
> 
>   (A) Para construcoes via cortes de Dedekind: 
> 
>   Rudin, W.- Priciples of Real Analysis-3rd edition, McGraw Hill
> International Editions (da segunda edicao deste livro ha', infelizmente,
> uma traducao, mas essa edicao nao tras a cosntrucao de R). Veja o apendice
> ao capitulo 1.
> 
>   Spivak, M.- Calculus (vol II) - Editorial Reverte, veja o apendice 1 
> deste texto.
> 
>   (B) Para construcoes atraves de sequencias de Cauchy (particularmente
> gosto mais deste enfoque):
> 
>   Esta' feita na seccao 5 do livro
> 
>   Hewitt, E. & Stromberg, K.- Real and Abstract Analysis - Springer.
>   
> Um ultimo comentario, historico, a construcao por cortes e' devida a
> Dedekind (foi publicada pela promeira vez em 1872) e por sequencias de
> Cauchy e' devida a Cantor, sendo originariamente publicada tambem em 1872.

Outra constru,c~ao est a de Conway, como um caso particular da de n'umeros
surreais, veja "On Numbers and Games", John H. Conway.

[]s, N.
=
Instruções para entrar na lista, sair da lista e usar a lista em
http://www.mat.puc-rio.br/~nicolau/olimp/obm-l.html
=


[obm-l] Resultado do Brasil na IMO

2003-07-15 Por tôpico Nicolau C. Saldanha
O resultado do Brasil na IMO em termos de pontos por problema segue abaixo.
O resultado por medalhas sai dentro de algumas horas. Digamos que j'a tivemos
resultados melhores e piores e devemos sempre apoiar os nossos atletas.

  1 2 3 4 5 6 Total

BRA1 = Alex   7 3 0 7 1 0  18
BRA2 = Samuel 2 2 0 7 0 7  18
BRA3 = Rafael 0 1 0 7 3 0  11
BRA4 = Larissa1 0 0 7 0 1   9
BRA5 = Fabio  7 3 0 7 1 1  19
BRA6 = David  2 3 0 7 4 1  17

[]s, Nicolau
=
Instruções para entrar na lista, sair da lista e usar a lista em
http://www.mat.puc-rio.br/~nicolau/olimp/obm-l.html
=


Re: [obm-l]

2003-07-15 Por tôpico Rafael Ando
5exp(2003) eh 5*e^2003, sendo exp(n) = e^n


From: Eduardo Henrique Leitner <[EMAIL PROTECTED]>
Reply-To: [EMAIL PROTECTED]
To: [EMAIL PROTECTED]
Subject: Re: [obm-l]
Date: Wed, 16 Jul 2003 01:35:43 -0300
o q significa 5exp(2003)?

On Wed, Jul 16, 2003 at 12:46:20AM -0300, Frederico Reis Marques de Brito 
wrote:
> Aexpressão é igual a 5exp(2003) / 5exp(2001)  = exp(2003)/exp(2001)= 
exp(2)
> . Desde que  e é aproximadamente igual a 2,7, decorre que a parte 
inteira
> de e^2 = 7.  Logo, 7 é o maior inteiro que não supera o número dado pela
> expressão.
>
>
> >From: [EMAIL PROTECTED]
> >Reply-To: [EMAIL PROTECTED]
> >To: [EMAIL PROTECTED]
> >Subject: [obm-l] Date: Tue, 15 Jul 2003 23:55:56 -0300
> >
> >Alguém me ajude com essa questão:
> >Qual é o maior valor inteiro que não supera o número:
> >( 2exp(2003)+3exp(2003)/(2exp(2001)+3exp(2001))
> >
> >_
> >Voce quer um iGMail protegido contra vírus e spams?
> >Clique aqui: http://www.igmailseguro.ig.com.br
> >Ofertas imperdíveis! Link: http://www.americanas.com.br/ig/
> >Ofertas imperdíveis!
> >
> 
>=
> >Instruções para entrar na lista, sair da lista e usar a lista em
> >http://www.mat.puc-rio.br/~nicolau/olimp/obm-l.html
> 
>=
>
> _
> MSN Hotmail, o maior webmail do Brasil.  http://www.hotmail.com
>
> 
=
> Instruções para entrar na lista, sair da lista e usar a lista em
> http://www.mat.puc-rio.br/~nicolau/olimp/obm-l.html
> 
=
=
Instruções para entrar na lista, sair da lista e usar a lista em
http://www.mat.puc-rio.br/~nicolau/olimp/obm-l.html
=
_
MSN Hotmail, o maior webmail do Brasil.  http://www.hotmail.com
=
Instruções para entrar na lista, sair da lista e usar a lista em
http://www.mat.puc-rio.br/~nicolau/olimp/obm-l.html
=


[obm-l] Re: [obm-l] Conjuntos numeráveis e conjuntos não enumerávei

2003-07-15 Por tôpico Nicolau C. Saldanha
On Wed, Jul 16, 2003 at 02:24:02AM -0300, Artur Costa Steiner wrote:
> Bom dia a todos, Eu gostaria de levantar um assunto que há algum tempo me
> intriga. O fato de um conjunto ser ou nao numeravel eh algo intrinseco ao
> conjunto ou depende da topologia nele definida?

O conceito de cardinal 'e um dos poucos que est intrinseco ao conjunto, n~ao
depende de nenhum outro tipo de estrutura, seja topol'ogica ou qq outra.

>  Vou tentar explicar porque isso me
>  intriga. Para tanto, consideremos o
>  conjunto R dos reais com a topologia
>  usual, definida pela metrica Euclidiana.
>  Nesta situacao, sabemos que R eh
>  completo. Vamos agora analisar uma prova
>  classica de que R nao eh numeravel.
>  Tomemos entao o intervalo I = [0,1] e
>  seja X ={x_1,...  x_k...} uma enumeracao
>  de elementos de I. Como todo elemento de
>  R eh ponto de acumulacao do mesmo,
>  podemos escolher um subintervalo fechado
>  I_1 de I que nao contenha x_1. Da mesma
>  forma, podemos escolher um subintervalo
>  fechado I_2 de I_1 que nao contenha x_2.
>  Atraves de um raciocinio indutivo,
>  constatamos que este processo gera uma
>  sequencia {I_k} de subintervalos
>  fechados de I tal que, para cada k, x_k
>  nao pertence a I_k. Logo, nenhum
>  elemento da enumeracao X eh comum a
>  todos os intervalos I_k. Mas como R eh
>  completo, existe, segundo um conhecido
>  teorema, um elemento x comum a todos os
>  I_k que, consequentemente, pertence a I
>  mas nao estah englobado na enumeracao X.
>  Isto nos mostra que nenhuma enumeracao
>  de elementos de I cobre a totalidade de
>  I, do que deduzimos que I e, portanto, o
>  proprio R, nao sao numeraveis. 
> 
> Mas para que esta prova seja valida, precisamos saber previamente que R eh
> completo e que todos seus elementos sao pontos de acumulacao do mesmo. Tais
> condicoes dependem da topologia definida em R.

Sim, a prova que voc^e acabou de dar depende da topologia de R.  Ali'as depende
tamb'em da ordem. Mas n~ao importa, R continua a ser n~ao enumer'avel mesmo sem
esta estrutura (a demonstra,c~ao pode n~ao funcionar, mas a conclus~ao ainda 'e
correta).
>  Se, por exemplo,
>  tivermos R
>  estruturado com
>  a topologia
>  definida pela
>  metrica
>  discreta, entao
>  a prova acima
>  deixa de valer.
>  Embora R
>  continue sendo
>  completo (as
>  sequencias de
>  Cauchy passam a
>  ser aquelas que
>  se tornam
>  constantes a
>  partir de algum
>  k), nenhum
>  elemento de R,
>  na metrica
>  discreta, eh
>  ponto de
>  

Re: [obm-l] polinomios

2003-07-15 Por tôpico Eduardo Casagrande Stabel
Oi chará!

O polinômio Q(x) = P(x) - 1 é de grau 5 e tem como raízes {1, 2, 3, 4, 5},
portanto Q(x) = A(x - 1)(x - 2)(x - 3)(x - 4)(x - 5), para algum A. Sabemos
que Q(6) = P(6) - 1 = - 1 = A*5!, logo A = -1/5!. O valor de P(0) = Q(0) + 1
= (-1)(-2)(-3)(-4)(-5)/(-5!) + 1 = 2. Portanto P(0) = 2. Acho que é isto.

Abraço!
Duda.


From: "Eduardo Henrique Leitner" <[EMAIL PROTECTED]>
> Fundamentos de Matemática Elementar, volume 6
>
> Gelson Iezzi
>
> 145. Seja P(x) um polinômio de 5^o grau que satisfaz as condições:
>
> 1 = P(1) = P(2) = P(3) = P(4) = P(5)
> e
> 0 = P(6).
>
> Qual o valor de P(0)?


=
Instruções para entrar na lista, sair da lista e usar a lista em
http://www.mat.puc-rio.br/~nicolau/olimp/obm-l.html
=


[obm-l] Conjuntos numeráveis e conjuntos não enumeráveis

2003-07-15 Por tôpico Artur Costa Steiner
Bom dia a todos,
Eu gostaria de levantar um assunto que há algum tempo me intriga. O fato
de um conjunto ser ou nao numeravel eh algo intrinseco ao conjunto ou
depende da topologia nele definida? Vou tentar explicar porque isso me
intriga. Para tanto, consideremos o conjunto R dos reais com a topologia
usual, definida pela metrica Euclidiana. Nesta situacao, sabemos que R
eh completo. Vamos agora analisar uma prova classica de que R nao eh
numeravel. Tomemos entao o intervalo I = [0,1] e seja X ={x_1,...
x_k...} uma enumeracao de elementos de I. Como todo elemento de R eh
ponto de acumulacao do mesmo, podemos escolher um subintervalo fechado
I_1 de I que nao contenha x_1. Da mesma forma, podemos escolher um
subintervalo fechado I_2 de I_1 que nao contenha x_2. Atraves de um
raciocinio indutivo, constatamos que este processo gera uma sequencia
{I_k} de subintervalos fechados de I tal que, para cada k, x_k nao
pertence a I_k. Logo, nenhum elemento da enumeracao X eh comum a todos
os intervalos I_k. Mas como R eh completo, existe, segundo um conhecido
teorema, um elemento x comum a todos os I_k que, consequentemente,
pertence a I mas nao estah englobado na enumeracao X. Isto nos mostra
que nenhuma enumeracao de elementos de I cobre a totalidade de I, do que
deduzimos que I e, portanto, o proprio R, nao sao numeraveis. 

Mas para que esta prova seja valida, precisamos saber previamente que R
eh completo e que todos seus elementos sao pontos de acumulacao do
mesmo. Tais condicoes dependem da topologia definida em R. Se, por
exemplo, tivermos R estruturado com a topologia definida pela metrica
discreta, entao a prova acima deixa de valer. Embora R continue sendo
completo (as sequencias de Cauchy passam a ser aquelas que se tornam
constantes a partir de algum k), nenhum elemento de R, na metrica
discreta, eh ponto de acumulacao do mesmo. Logo, os requisitos basicos
para a prova mencionada nao mais vigoram.

Na realidade, a prova que reproduzi para I eh um caso particular de uma
outra que diz que se um espaco topologico S eh compacto e nao contem
pontos isolados, entao S nao eh numeravel. O que acarreta que se um
espaco topologico contem um sub-espaco com as caracteristicas de S,
entao o espaco nao eh numeravel. Mas, os conceitos de conjunto compacto
e de pontos isolados dependem da topologia definida.

Uma outra prova da nao enumerabilidade de R eh a de Cantor, baseada em
expansoes decimais dos numeros reais. Mas esta prova tambem pressupoem R
com a topologia usual.

E eh isto que me intriga. Eh posivel provar que R nao eh numeravel sem
admitirmos alguma topologia nele definida? O fato de provarmos que um
conjunto eh ou nao numeravel numa topologia garante que tais condicoes
sao preservadas em qualquer topologia? 

Um abraco
Artur

=
Instruções para entrar na lista, sair da lista e usar a lista em
http://www.mat.puc-rio.br/~nicolau/olimp/obm-l.html
=


Re: [obm-l]

2003-07-15 Por tôpico Eduardo Henrique Leitner
o q significa 5exp(2003)?

On Wed, Jul 16, 2003 at 12:46:20AM -0300, Frederico Reis Marques de Brito wrote:
> Aexpressão é igual a 5exp(2003) / 5exp(2001)  = exp(2003)/exp(2001)= exp(2) 
> . Desde que  e é aproximadamente igual a 2,7, decorre que a parte inteira 
> de e^2 = 7.  Logo, 7 é o maior inteiro que não supera o número dado pela 
> expressão.
> 
> 
> >From: [EMAIL PROTECTED]
> >Reply-To: [EMAIL PROTECTED]
> >To: [EMAIL PROTECTED]
> >Subject: [obm-l] Date: Tue, 15 Jul 2003 23:55:56 -0300
> >
> >Alguém me ajude com essa questão:
> >Qual é o maior valor inteiro que não supera o número:
> >( 2exp(2003)+3exp(2003)/(2exp(2001)+3exp(2001))
> >
> >_
> >Voce quer um iGMail protegido contra vírus e spams?
> >Clique aqui: http://www.igmailseguro.ig.com.br
> >Ofertas imperdíveis! Link: http://www.americanas.com.br/ig/
> >Ofertas imperdíveis!
> >
> >=
> >Instruções para entrar na lista, sair da lista e usar a lista em
> >http://www.mat.puc-rio.br/~nicolau/olimp/obm-l.html
> >=
> 
> _
> MSN Hotmail, o maior webmail do Brasil.  http://www.hotmail.com
> 
> =
> Instruções para entrar na lista, sair da lista e usar a lista em
> http://www.mat.puc-rio.br/~nicolau/olimp/obm-l.html
> =
=
Instruções para entrar na lista, sair da lista e usar a lista em
http://www.mat.puc-rio.br/~nicolau/olimp/obm-l.html
=


Re: [obm-l]

2003-07-15 Por tôpico Frederico Reis Marques de Brito
Aexpressão é igual a 5exp(2003) / 5exp(2001)  = exp(2003)/exp(2001)= exp(2) 
. Desde que  e é aproximadamente igual a 2,7, decorre que a parte inteira de 
 e^2 = 7.  Logo, 7 é o maior inteiro que não supera o número dado pela 
expressão.


From: [EMAIL PROTECTED]
Reply-To: [EMAIL PROTECTED]
To: [EMAIL PROTECTED]
Subject: [obm-l] Date: Tue, 15 Jul 2003 23:55:56 -0300
Alguém me ajude com essa questão:
Qual é o maior valor inteiro que não supera o número:
( 2exp(2003)+3exp(2003)/(2exp(2001)+3exp(2001))
_
Voce quer um iGMail protegido contra vírus e spams?
Clique aqui: http://www.igmailseguro.ig.com.br
Ofertas imperdíveis! Link: http://www.americanas.com.br/ig/
Ofertas imperdíveis!
=
Instruções para entrar na lista, sair da lista e usar a lista em
http://www.mat.puc-rio.br/~nicolau/olimp/obm-l.html
=
_
MSN Hotmail, o maior webmail do Brasil.  http://www.hotmail.com
=
Instruções para entrar na lista, sair da lista e usar a lista em
http://www.mat.puc-rio.br/~nicolau/olimp/obm-l.html
=


Re: [obm-l]

2003-07-15 Por tôpico A. C. Morgado
( 2exp(2003)+3exp(2003)/(2exp(2001)+3exp(2001)) = 5exp(2003)/5exp(2001) = exp(2) 
aproximadamente
2,7^2 = 7,3
A resposta eh 7.


[EMAIL PROTECTED] wrote:

Alguém me ajude com essa questão: 
Qual é o maior valor inteiro que não supera o número: 
( 2exp(2003)+3exp(2003)/(2exp(2001)+3exp(2001)) 

_
Voce quer um iGMail protegido contra vírus e spams?
Clique aqui: http://www.igmailseguro.ig.com.br
Ofertas imperdíveis! Link: http://www.americanas.com.br/ig/
Ofertas imperdíveis!
=
Instruções para entrar na lista, sair da lista e usar a lista em
http://www.mat.puc-rio.br/~nicolau/olimp/obm-l.html
=
 

=
Instruções para entrar na lista, sair da lista e usar a lista em
http://www.mat.puc-rio.br/~nicolau/olimp/obm-l.html
=


[obm-l]

2003-07-15 Por tôpico luiz-ernesto
Alguém me ajude com essa questão: 
Qual é o maior valor inteiro que não supera o número: 
( 2exp(2003)+3exp(2003)/(2exp(2001)+3exp(2001)) 

_
Voce quer um iGMail protegido contra vírus e spams?
Clique aqui: http://www.igmailseguro.ig.com.br
Ofertas imperdíveis! Link: http://www.americanas.com.br/ig/
Ofertas imperdíveis!

=
Instruções para entrar na lista, sair da lista e usar a lista em
http://www.mat.puc-rio.br/~nicolau/olimp/obm-l.html
=


[obm-l] Re: [obm-l] Re: [obm-l] Re: Como os Matemáticos Complicam II

2003-07-15 Por tôpico Nicolau C. Saldanha
On Wed, Jul 09, 2003 at 05:57:13PM -0300, J.Paulo roxer ´til the end wrote:
> Não está sendo muito produtiva esta lista pra mim.Entendo muito pouco.

Eu me arrependo profundamente de ter permitido que esta pessoa voltasse
a se inscrever na lista. Por favor vah embora e n~ao volte a escrever
para esta lista nunca mais, nem com este e-mail nem com nenhum outro.

Estou escrevendo da IMO. Os resultados devem sair dentro de algumas horas.

[]s, N.
=
Instruções para entrar na lista, sair da lista e usar a lista em
http://www.mat.puc-rio.br/~nicolau/olimp/obm-l.html
=


Re: [obm-l] polinomios

2003-07-15 Por tôpico Eduardo Henrique Leitner
hahaha, mas você usou calculadora né?!?!

orra, se você fez tudo aquilo na mão cara... eu te respeito! (ou não)

mas valeu ae

On Wed, Jul 16, 2003 at 01:36:59AM +0200, Fernando Henrique Ferraz Pereira da Rosa 
wrote:
>  Incidentalmente há uns 2 anos também 'trombei' com esse problema e tive a
> paciência de resolver o sistema para achar a resposta. De qualquer forma
> postei uma mensagem para a lista e me mandaram uma sugestão mais simples:
>  http://www.mat.puc-rio.br/~nicolau/olimp/obm-l.200107/msg4.html
> 
> 
> -- 
> []'s
> 
> +++ GMX - Mail, Messaging & more  http://www.gmx.net +++
> 
> Jetzt ein- oder umsteigen und USB-Speicheruhr als Prämie sichern!
> 
> =
> Instruções para entrar na lista, sair da lista e usar a lista em
> http://www.mat.puc-rio.br/~nicolau/olimp/obm-l.html
> =
=
Instruções para entrar na lista, sair da lista e usar a lista em
http://www.mat.puc-rio.br/~nicolau/olimp/obm-l.html
=


Re: [obm-l] Transformada de Laplace

2003-07-15 Por tôpico Marcus Alexandre Nunes



Bah, nem tinha me ligado em usar estas 
propriedades trigonométricas. Valeu Leandro.
 
Marcus Alexandre Nunes[EMAIL PROTECTED]http://darwingauss.blogspot.comUIN 
114153703


Re: [obm-l] polinomios

2003-07-15 Por tôpico Eduardo Henrique Leitner
valeu cara! eu realmente nao hia pensar nisso...

On Tue, Jul 15, 2003 at 07:46:29AM -0300, Marcelo Rufino de Oliveira wrote:
> Seja Q(x) = P(x) - 1   =>   Q(1) = Q(2) = Q(3) = Q(4) = Q(5) = 0. Como Q(x)
> também possui grau cinco, 1, 2, 3, 4, e 5 são as cinco raízes de Q(x)   =>
> Q(x) = A(x - 1)(x - 2)(x - 3)(x - 4)(x - 5) (1)
> 
> P(6) = 0   =>   Q(6) = - 1
> 
> Aplicando x = 6 em (1)   =>   - 1 = A.5.4.3.2.1   =>   A = - 1/120   =>
> Q(x) = - (x - 1)(x - 2)(x - 3)(x - 4)(x - 5)/120   =>
> P(x) = 1 - (x - 1)(x - 2)(x - 3)(x - 4)(x - 5)/120
> 
> Assim,  P(0) = 1 - (-1)(-2)(-3)(-4)(-5)/120   =>   P(0) = 2
> 
> 
> 
> Marcelo Rufino de Oliveira
> 
> 
> - Original Message -
> From: "Eduardo Henrique Leitner" <[EMAIL PROTECTED]>
> To: "lista de matemática" <[EMAIL PROTECTED]>
> Sent: Tuesday, July 15, 2003 6:52 PM
> Subject: [obm-l] polinomios
> 
> 
> > Fundamentos de Matemática Elementar, volume 6
> >
> > Gelson Iezzi
> >
> > 145. Seja P(x) um polinômio de 5^o grau que satisfaz as condições:
> >
> > 1 = P(1) = P(2) = P(3) = P(4) = P(5)
> > e
> > 0 = P(6).
> >
> > Qual o valor de P(0)?
> >
> >
> >
> > eu tentei fazer pelo sistema... mas putz... sem condições...
> > =
> > Instruções para entrar na lista, sair da lista e usar a lista em
> > http://www.mat.puc-rio.br/~nicolau/olimp/obm-l.html
> > =
> >
> =
> Instruções para entrar na lista, sair da lista e usar a lista em
> http://www.mat.puc-rio.br/~nicolau/olimp/obm-l.html
> =
=
Instruções para entrar na lista, sair da lista e usar a lista em
http://www.mat.puc-rio.br/~nicolau/olimp/obm-l.html
=


Re: [obm-l] Matrizes

2003-07-15 Por tôpico A. C. Morgado
Traço AB = traço BA
traço (AB-BA)=0
traço I = n
[EMAIL PROTECTED] wrote:

Prove que não existem matrizes reais A e B tal que AB-BA=I

"Mathematicus nascitur, non fit"
Matemáticos não são feitos, eles nascem
---
Gabriel Haeser
www.gabas.cjb.net
--
Use o melhor sistema de busca da Internet
Radar UOL - http://www.radaruol.com.br


=
Instruções para entrar na lista, sair da lista e usar a lista em
http://www.mat.puc-rio.br/~nicolau/olimp/obm-l.html
=
 

=
Instruções para entrar na lista, sair da lista e usar a lista em
http://www.mat.puc-rio.br/~nicolau/olimp/obm-l.html
=


Re: [obm-l] Problemas da IMO

2003-07-15 Por tôpico Marcio Afonso A. Cohen
Realmente, sua solucao me parece perfeita.. Alem de nao usar que o
quadrilatero eh inscritivel.. legal.
Voce pensou nos outros? Pensei bem no 2 e no 3, mas nao consegui fechar
nenhum.. O 3 eu acredito que seja alguma desigualdade virando igualdade, e
quero tentar mais pra ver se da certo..

- Original Message -
From: <[EMAIL PROTECTED]>
To: <[EMAIL PROTECTED]>
Sent: Tuesday, July 15, 2003 12:05 PM
Subject: Re: [obm-l] Problemas da IMO


> Marcio,
> achei legal essa sua solucao por complexos. Uma outra solucao
> trivial (e acho que a de 99% dos participantes) seria a seguinte:
>
> quad. APDR inscritivel  =>  PR = AD.sen( quad. CQRD inscritivel  =>  RQ = DC.sen(
> PR = RQ  =>  AD/DC = sen(
> Sendo S e T os pontos de interseccao das bissetrizes internas dos
> angulos 
> AS/SC  =  AB/BC  =  AD/DC  =  AT/TC   Logo, S = T
>   (1)   (2)   (3)
>
> (1) e (3) - teorema da bissetriz interna
> (2) - por (*)
>
> abracos,
>
> #
> # MSc. Edson Ricardo de A. Silva#
> # Computer Graphics Group (CRAB)#
> # Federal University of Ceara (UFC) #
> #
>
> On Tue, 15 Jul 2003, Marcio Afonso A. Cohen wrote:
>
> > Eu sei que ninguem gosta muito disso, mas esse problema 4 (que eu
ateh
> > imagino que nao seja dificil por plana) eh bem simples na conta bruta..
Eh
> > impressionante como complexos ajudam nos problemas de geometria da imo..
> > aquele artigo da eureka 6 eh realmente muito util!
> >
> > Coloque o circuncentro na origem, e represente os vertices pelos
> > complexos a,b,c,d, todos de modulo 1u.m.
> > Reta ab: z+abz' = a+b
> > Reta perpendicular a ab passando por d: z-abz'=d-abd'
> > Logo, o ponto P eh 2p = [a+b+d-ab/d]
> > Portanto, 2q = [a+c+d-ac/d] e 2r = [b+c+d-bc/d].
> > Como p,q,r sao colineares (reta de simpson), e |p-q| = |q-r|:
> > p-q = q-r, ou seja: b-c + ac/d - ab/d = a-b +bc/d-ac/d
> > Arrumando: (b-c) - (a/d)(b-c) = (a-b) - (c/d)(a-b) sse
(b-c)(d-a)=(a-b)(d-c)
> > Tirando modulo, isso significa que BC*AD = AB*DC. E isso fecha o
problema.
> > De fato, sendo I o peh da bissetriz de ABC em AC, entao, AI/IC = AB/BC e
vc
> > quer provar que I eh peh da bissetriz de ADC, i.e, que AI/IC=AD/DC
(teorema
> > da bissetriz interna, ida e volta). Portanto, eh suficiente provar que
AB*DC
> > = AD*BC.
> >
> > Vou pensar nos outros agora, esse foi o que eu achei que seria mais
> > facil.. (ja pensei no 2 e no 1 um pouco tmb..)
> >
> >
> > - Original Message -
> > From: <[EMAIL PROTECTED]>
> > To: <[EMAIL PROTECTED]>
> > Cc: <[EMAIL PROTECTED]>; <[EMAIL PROTECTED]>
> > Sent: Monday, July 14, 2003 3:38 PM
> > Subject: [obm-l] Problemas da IMO
> >
> >
> > >
> > >
> > > Prova da IMO retirada do Site http://www.mathlinks.go.ro/
> > >
> > > O Problema 1 é nois que mandou...
> > >
> > >
> > > First Day - 44th IMO 2003 Japan
> > >
> > > 1. Let A be a 101-element subset of the set S={1,2,3,...,100}.
Prove
> > that
> > > there exist numbers t_1, t_2, ..., t_{100} in S such that the sets
> > >
> > > Aj = { x + tj | x is in A } for each j = 1, 2, ..., 100
> > >
> > > are pairwise disjoint.
> > >
> > >
> > > 2. Find all pairs of positive integers (a,b) such that the number
> > >
> > > a^2 / ( 2ab^2-b^3+1) is also a positive integer.
> > >
> > > 3. Given is a convex hexagon with the property that the segment
connecting
> > the
> > > middle points of each pair of opposite sides in the hexagon is
sqrt(3) /
> > 2
> > > times the sum of those sides' sum.
> > >
> > > Prove that the hexagon has all its angles equal to 120.
> > >
> > >
> > > Second Day - 44th IMO 2003 Japan
> > >
> > > 4. Given is a cyclic quadrilateral ABCD and let P, Q, R be feet of the
> > > altitudes from D to AB, BC and CA respectively. Prove that if PR = RQ
then
> > the
> > > interior angle bisectors of the angles < ABC and < ADC are concurrent
on
> > AC.
> > >
> > > 5. Let x1 <= x2 <= ... <= xn be real numbers, n>2.
> > >
> > > a) Prove the following inequality:
> > >
> > > (sum  ni,j=1 | xi - xj | ) 2 <= 2/3 ( n^2 - 1 )sum ni,j=1 ( xi - xj)^2
> > >
> > > b) Prove that the equality in the inequality above is obtained if and
only
> > if
> > > the sequence (xk) is an arithemetical progression.
> > >
> > > 6. Prove that for each given prime p there exists a prime q such that
> > n^p - p
> > > is not divisible by q for each positive integer n.
> > >
> > >
> >
> >
=
> > Instruções para entrar na lista, sair da lista e usar a lista em
> > http://www.mat.puc-rio.br/~nicolau/olimp/obm-l.html
> >
=
> >
> =
> Instruções para entrar na lista, sair da lista e usar a lista em
> http://www.mat.puc-rio.br/~nicolau/olimp/obm-l.html
> =
>

==

[obm-l] Matrizes

2003-07-15 Por tôpico ghaeser
Prove que não existem matrizes reais A e B tal que AB-BA=I

"Mathematicus nascitur, non fit"
Matemáticos não são feitos, eles nascem
---
Gabriel Haeser
www.gabas.cjb.net


--
Use o melhor sistema de busca da Internet
Radar UOL - http://www.radaruol.com.br



=
Instruções para entrar na lista, sair da lista e usar a lista em
http://www.mat.puc-rio.br/~nicolau/olimp/obm-l.html
=


Re: [obm-l] polinomios

2003-07-15 Por tôpico Fernando Henrique Ferraz Pereira da Rosa
 Incidentalmente há uns 2 anos também 'trombei' com esse problema e tive a
paciência de resolver o sistema para achar a resposta. De qualquer forma
postei uma mensagem para a lista e me mandaram uma sugestão mais simples:
 http://www.mat.puc-rio.br/~nicolau/olimp/obm-l.200107/msg4.html


-- 
[]'s

+++ GMX - Mail, Messaging & more  http://www.gmx.net +++

Jetzt ein- oder umsteigen und USB-Speicheruhr als Prämie sichern!

=
Instruções para entrar na lista, sair da lista e usar a lista em
http://www.mat.puc-rio.br/~nicolau/olimp/obm-l.html
=


RE: [obm-l] Transformada de Laplace

2003-07-15 Por tôpico Leandro Lacorte Recôva









Porque voce nao usa as
formulas : 

 

Sin(a + b) =
sin(a).cos(b) + cos(a)sin(b)

Sin(a – b) =
sin(a).cos(b) – cos(b)sin(a) 

 

Some (1) e (2)

 

Sin(a+b) + sin(a-b) =
2.sin(a).cos(b)  

 

Portanto, g(t) =  cos(mt).sin(nt)
= (sin(m+n)t + sin(m-n)t)/2 

 

Assim, sabendo que a
transformada de sin(at) = (a)/(s^2+a^2) entao,

 

G(s) = ((m+n)/(s^2+(m+n)^2)
+ (m-n)/(s^2+(m-n)^2))*1/2   (Lembre que a transformada e linear e tirei a
transformada de cada membro de g(t).

 

Se errei algum sinal,
pode corrigir, mas o raciocinio e esse. 

 

Leandro

Los Angeles, CA

 

-Original Message-
From:
[EMAIL PROTECTED] [mailto:[EMAIL PROTECTED] On Behalf Of Marcus Alexandre Nunes
Sent: Tuesday, July
 15, 2003 3:45 PM
To: Lista OBM
Subject: [obm-l] Transformada de
Laplace

 



Como fazer para calcular as trnasformadas de Laplace das
funções





 





cos(mt)sen(nt)





 





e





 





exp(-2x)(cosx)^2 ?





 






Marcus Alexandre Nunes
[EMAIL PROTECTED]
http://darwingauss.blogspot.com
UIN 114153703










[obm-l] Problemas IMO - Questao 4

2003-07-15 Por tôpico Rodrigo Villard Milet
Parece estar certo... Eu fiz uma solução legalzinha... segue :



Deixo um espaço em branco...





.








.

















Veja que podemos supor que P está fora de AB e Q está dentro de BC, pois
como A+C= 180, um dos A ou C deve ser agudo e o outro obtuso. [XYZ] = área
do triângulo XYZ.
Veja que [ACQ]=[APC] (pois como PR=QR, então [APR]=[ARQ] e [CPR]=[CRQ], já
que P,Q e R são colineares - reta de Simson)
Logo [BCP]-[ABC]=[ABC]-[ABQ], então BP.BC-BA.BC=BA.BC-BA.BQ, o que é
equivalente a BA.(BC-BQ) = BC.(BP-BA) ... BA.CQ=BC.PA. Como PA/CQ =
AD.cosC/CD.cosC  = AD/CD, segue que BA/BC=AD/CD, o que finaliza o problema.

Abraços,
 Villard
-Mensagem original-
De: [EMAIL PROTECTED] <[EMAIL PROTECTED]>
Para: [EMAIL PROTECTED] <[EMAIL PROTECTED]>
Data: Terça-feira, 15 de Julho de 2003 13:00
Assunto: [obm-l] Problemas IMO - Questao 4


>So um pequeno detalhe... nao precisei usar o fato de ABCD ser incritivel
>(pelo menos nao explicitamente). Alguem poderia comentar isso?
>
>#
># MSc. Edson Ricardo de A. Silva#
># Computer Graphics Group (CRAB)#
># Federal University of Ceara (UFC) #
>#
>
>> achei legal essa sua solucao por complexos. Uma outra solucao
>> trivial (e acho que a de 99% dos participantes) seria a seguinte:
>>
>> quad. APDR inscritivel  =>  PR = AD.sen(> quad. CQRD inscritivel  =>  RQ = DC.sen(>
>> PR = RQ  =>  AD/DC = sen(>
>> Sendo S e T os pontos de interseccao das bissetrizes internas dos
>> angulos >
>> AS/SC  =  AB/BC  =  AD/DC  =  AT/TC   Logo, S = T
>>   (1)   (2)   (3)
>>
>> (1) e (3) - teorema da bissetriz interna
>> (2) - por (*)
>>
>> abracos,
>=
>Instruções para entrar na lista, sair da lista e usar a lista em
>http://www.mat.puc-rio.br/~nicolau/olimp/obm-l.html
>=
>

=
Instruções para entrar na lista, sair da lista e usar a lista em
http://www.mat.puc-rio.br/~nicolau/olimp/obm-l.html
=


[obm-l] Desafio Dificio (raposas e galinhas)

2003-07-15 Por tôpico MuriloRFL



"Vc é capaz de distribuir em um tabuleiro 5x5, 5 
raposas e 3 galinhas de tal forma que nenhuma raposa ataque alguma das 3 
galinhas? Sabe-se que as raposas se movimentam como rainhas no 
jogo de xadrez."


[obm-l] Transformadas de Laplace II

2003-07-15 Por tôpico Marcus Alexandre Nunes



Eu esqueci de dizer no outro e-mail: se for 
possível colocar, de modo genérico, a regra ou propriedade que foi usada pra 
resolver o problema, eu agradeço muito.
 
Marcus Alexandre Nunes[EMAIL PROTECTED]http://darwingauss.blogspot.comUIN 
114153703


Re: [obm-l] polinomios

2003-07-15 Por tôpico Marcelo Rufino de Oliveira
Seja Q(x) = P(x) - 1   =>   Q(1) = Q(2) = Q(3) = Q(4) = Q(5) = 0. Como Q(x)
também possui grau cinco, 1, 2, 3, 4, e 5 são as cinco raízes de Q(x)   =>
Q(x) = A(x - 1)(x - 2)(x - 3)(x - 4)(x - 5) (1)

P(6) = 0   =>   Q(6) = - 1

Aplicando x = 6 em (1)   =>   - 1 = A.5.4.3.2.1   =>   A = - 1/120   =>
Q(x) = - (x - 1)(x - 2)(x - 3)(x - 4)(x - 5)/120   =>
P(x) = 1 - (x - 1)(x - 2)(x - 3)(x - 4)(x - 5)/120

Assim,  P(0) = 1 - (-1)(-2)(-3)(-4)(-5)/120   =>   P(0) = 2



Marcelo Rufino de Oliveira


- Original Message -
From: "Eduardo Henrique Leitner" <[EMAIL PROTECTED]>
To: "lista de matemática" <[EMAIL PROTECTED]>
Sent: Tuesday, July 15, 2003 6:52 PM
Subject: [obm-l] polinomios


> Fundamentos de Matemática Elementar, volume 6
>
> Gelson Iezzi
>
> 145. Seja P(x) um polinômio de 5^o grau que satisfaz as condições:
>
> 1 = P(1) = P(2) = P(3) = P(4) = P(5)
> e
> 0 = P(6).
>
> Qual o valor de P(0)?
>
>
>
> eu tentei fazer pelo sistema... mas putz... sem condições...
> =
> Instruções para entrar na lista, sair da lista e usar a lista em
> http://www.mat.puc-rio.br/~nicolau/olimp/obm-l.html
> =
>
=
Instruções para entrar na lista, sair da lista e usar a lista em
http://www.mat.puc-rio.br/~nicolau/olimp/obm-l.html
=


[obm-l] Transformada de Laplace

2003-07-15 Por tôpico Marcus Alexandre Nunes



Como fazer para calcular as trnasformadas de 
Laplace das funções
 
cos(mt)sen(nt)
 
e
 
exp(-2x)(cosx)^2 ?
 
Marcus Alexandre Nunes[EMAIL PROTECTED]http://darwingauss.blogspot.comUIN 
114153703


Re: [obm-l] dificuldade

2003-07-15 Por tôpico Marcelo Rufino de Oliveira



 

  - Original Message - 
  From: 
  Fabrício 
  
  To: [EMAIL PROTECTED] 
  Sent: Tuesday, July 15, 2003 2:31 
PM
  Subject: [obm-l] dificuldade
  
  
  Prove que:se a,b e c são lados de um triângulo e 
  satisfaz a sentença a^2+b^2+c^2=9r^2, onde r é 
  o raio da circunferência circunscrita, então esse triângulo é 
  eqüilátero.
   
   
  1a. Solução (esperta):
  Usando o fato de que a distância entre circuncentro 
  O e o baricentro G de um triângulo é igual a R^2 - (a^2 + b^2 + c^2)/9 temos 
  que O = G e segue diretamente que o triângulo é 
  equilátero.
   
   
  2a. Solução (lusitana):
  Pela Lei dos Senos  a/R = 2.sen A,  b/R = 
  2.sen B  e  c/R = 2.sen C.
  Assim:  9 = 4.sen^2 A + 4.sen^2 B + 4.sen^2 C = 
  2 - 2.cos 2A + 2 - 2.cos 2B + 2 - cos 2C   
  =>
  cos 2A + cos 2B + cos 2C = - 
  3/2 agora é braço, use trigonometria para provar 
  que a única solução desta equação é A = B = C.
   
  Falou,
  Marcelo Rufino de Oliveira
   
   


[obm-l] polinomios

2003-07-15 Por tôpico Eduardo Henrique Leitner
Fundamentos de Matemática Elementar, volume 6

Gelson Iezzi

145. Seja P(x) um polinômio de 5^o grau que satisfaz as condições:

1 = P(1) = P(2) = P(3) = P(4) = P(5)
e
0 = P(6).

Qual o valor de P(0)?



eu tentei fazer pelo sistema... mas putz... sem condições...
=
Instruções para entrar na lista, sair da lista e usar a lista em
http://www.mat.puc-rio.br/~nicolau/olimp/obm-l.html
=


[obm-l] Seu crêisson!!! O_Erro_Comum...

2003-07-15 Por tôpico MuriloRFL



Joao Paulo,
 
Se vc nao entrou aki para ler coisas inuteis, 
eu tmb nao
E nem gasto vocabulario para 
responder tuas indagações (Talvez o Vocabulo Seu 
Crêisson!)...
Se vc sentiu-se ridicularizado é por que a 
carapulsa serviu...
Vai estudar... melhor do que vc 
ficar perdendo tempo respondendo piadinhas 
ridiculas...
O caracter ironico deste meu ridiculo ainda vai 
percurtir no seu ego.
Pq a verdade nao morre e nem nasce, ela não 
se cria, ela existe...
Quem disse que eu me preoucupo com a educação ou 
tenho um posicionamento sério para com os auto-excluidos e 
auto-incapacitados
 
Pra vc, vale a pena repetir:
 
"Quem 
conhece o outro é inteligente,
 Quem conhece a si mesmo 
é iluminado!
 Quem vence o outro é 
forte,
 Quem vence a si mesmo é 
invencível"
 
Mao-tse-tung
 
Sem mais, 
Indignado,
 
Murilo Lima
 
P.s - Pq seu e-mail é o unico cor de lilas 
gay?!
P.s2 - "Quem sabe faz a hora, nao espera 
acontecer" 
P.s3 - Pensava eu na minha ignorância, que esta 
lista fosse direcionada à OBM, não à "Como lecionar Matematica e outros bixos no 
ensino médio"
P.s4 - Me desculpem a todos da lista pelo tempo 
perdido neste e-mail...
 
 
- Original Message - 
From: J.Paulo roxer 
´til the end 
To: [EMAIL PROTECTED] 
Sent: Tuesday, July 15, 2003 4:13 PM
Subject: [obm-l] Re: [obm-l] Seu crêisson!!! 
O_Erro_Comum...

Vcs acham que esse é um posicionamento sério de 
quem gosta de matemática e se preocupa com a educação?
 
Murilo,não entrei aqui pra ler coisas inúteis 
nem estou disposto a perder meu tempo lendo piadinhas 
ridículas.
Vá engrandecer seu intelecto e dar respostas 
mais sábias quanto às minhas indagações.
 
João Paulo

  - Original Message - 
  From: 
  MuriloRFL 
  
  To: OBM-Lista 
  Sent: Tuesday, July 15, 2003 10:26 
  AM
  Subject: [obm-l] Seu crêisson!!! 
  O_Erro_Comum...
  
  Seu sinho JP,
   
   Nois, da 
  faucudadi seu Crêisson R$ ®, convidiamos vc pra nossia faucudadi 
  Nois temo aula especialica pra vc... Pra alunio reclamonico e griladiu!!! Temo 
  terapia de chókio A faucudade só necessitia di sua 
  assinaturia Prova nois num faiz pq nois num sabe mesmiu! nois 
  queremio é seu dinheirio E nem precisia escrivinha bestieiria!!! 
  Vc fiquia bem longi du lapisu.
   
          
  Gratilissimio,
   
     Faucudadi 
  seu Crêisson R$ ® 
   
  Agoria cum brilhiu nus olhius! 
  $-;-$
    
  X
   
   
  - Original Message - 
  From: J.Paulo 
  roxer ´til the end 
  To: [EMAIL PROTECTED] 
  Sent: Tuesday, July 15, 2003 1:55 AM
  Subject: [obm-l] Re: [obm-l] Re: [obm-l] 
  Re:_[obm-l]_O_Erro_Comum_da_Generalização_(_Ou_induções_equivocadas_)_
  
  Vc não se considera prepotente quando escreve coisas como "não admito que 
  NINGUÉM, especialmente um ignorante em Matemática " e "exijo que o sr 
  paulo recolha-se a sua insignificância "???
  Sou um inútil completo,não sei de nada,nunca 
  estudei,não estudo nem sirvo pra nada,correto?
  Certo,vc sabe de tudo,entende tudo que diz respeito à matemática,é o ser 
  supremo que merece a atenção de todos e merece ser muito respeitado.
   
  João Paulo
Email.it, the professional e-mail, 
gratis per te: clicca qui Sponsor:Iscriviti Gratis al primo corso 
on-line di Web Marketing in Italia! Collegati subito a: 
http://www.internet-marketing.it/?EMAILk0403aClicca qui 


[obm-l] Re: [obm-l] Análise Real

2003-07-15 Por tôpico Manuel Valentim Pera
Bernardo,

  Boa tarde,

  Só dois comentários:

  (1) Há algo "estranho" com o "corolário", ele é completamente trivial,
mas não sei como concluir do exercício original esse resultado. Veja o
seguinte, Q não pode ser a renuião enumerável de abertos, simplesmente
porque cada aberto não vazio de R contém um inervalo aberto (a,b) não
vazio. Logo se Q fosse uma reunião de abertos (enumerável ou não) Q
conteria (a,b). Iso é absurdo pois R-Q é denso em R. Talvez o corolário
seja "Q não é a intercecção enumerável de abertos". De fato isso segue-se
imediatamente do exercício proposto, por passagem ao complementar.

  (2) Não sei exatamente o contexto em que o exercício apareceu, às vezes
quando se fala em R, esconde-se quando se está usando Baire. Você precisa
saber alguma coisa, por exemplo que R não pode ser escrito como reunião
enumerável de fechados sem interior [pode chamar isso propriedade de Baire
da reta] ou algo equivalente para fazer o exercício (o que foi usado na
demonstração do outro email foi algo equivalente). Se você souber dessa
propriedade que enunciei acima, uma demonstração "alternativa" (que, no
fundo é exatamente igual) é a seguinte.

Suponha, por absurdo, que existem subconjuntos fechados de R, F_1,
F_2,..., F_n,... tais que a reunião de todos os F_n seja R-Q.

Como Q não tem interior (pois nenhum intervalo aberto da reta, não
vazio, está contido em Q) segue-se que cada F_n tem interior vazio.

   Qomo Q é enumerável tome {q_k, k em N} uma enumeração de Q e defina
T_j={q_j}, j=1,2,...

  Claro que cada T_j é fechado e de interior vazio.

  Então R = (R-Q) U Q seria a reunião dos F_n com os T_j. Então ter-se-ia
escrito R como uma reunião enumerável de fechados sem interior, o que
contraria a aupramencionada propriedade de Baire da reta.

Manuel Garcia


 On Tue, 15 Jul 2003 [EMAIL PROTECTED] wrote:

> Manuel,
> Boa tarde.
>
> Muito boa a solução para este problema, mas eu não conheço o teorema de
> Baire, nem lembro muito bem o que era um espaço de Baire. Mas o pior é que
> este problema tinha um "corolário": conclua que Q não é a reunião enumerável
> de abertos... então eu suponho que deve haver outro meio para resolver este
> problema. Para ser mais completo, deixo agora a referência:
> Curso de Análise, vol 1 - Elon Lages Lima
> Capítulo 5 (Topologia da Reta) - exercício 55
>
> Muito obrigado pela atenção,
> Bernardo
>

=
Instruções para entrar na lista, sair da lista e usar a lista em
http://www.mat.puc-rio.br/~nicolau/olimp/obm-l.html
=


[obm-l] curvas

2003-07-15 Por tôpico Marcelo Souza
Gente,
alguém pode me ajudar a resolver os problemas
1. Sejam e uma elipse e h uma hiperbole tendo focos em comum. Mostre que e e 
h se cortam perpedicularmente.
(alguém podria exibir uma solução usando derivadas, é pq estou tentando e 
não consegui)

2.Seja c:I->R^2 uma curva com segunda derivada e tq c'(t) dif de  (0,0) para 
todo t. Suponha ainda que a aceleração normal não se anule. A evoluta de c é 
a curva
a:I->R^2
 t --> (x(t),y(t))+(1/k(t)(x'(t)^2+y'(t)^2)^(1/2))*(-y'(t),x'(t)),
onde c(t)=(x(t),y(t)) e k(t) = (-y'x''+x'y'')/(x'^2+y'^2)^(3/2) (omiti t)

I) Mostre que a reta tangente a a em a(t) coincide com a normal a c em c(t).

valeu
[]'s, Marcelo
_
MSN Messenger: instale grátis e converse com seus amigos. 
http://messenger.msn.com.br

=
Instruções para entrar na lista, sair da lista e usar a lista em
http://www.mat.puc-rio.br/~nicolau/olimp/obm-l.html
=


[obm-l] Re: [obm-l] Análise Real

2003-07-15 Por tôpico bmat
Manuel,
Boa tarde.

Muito boa a solução para este problema, mas eu não conheço o teorema de
Baire, nem lembro muito bem o que era um espaço de Baire. Mas o pior é que
este problema tinha um "corolário": conclua que Q não é a reunião enumerável
de abertos... então eu suponho que deve haver outro meio para resolver este
problema. Para ser mais completo, deixo agora a referência:
Curso de Análise, vol 1 - Elon Lages Lima
Capítulo 5 (Topologia da Reta) - exercício 55

Muito obrigado pela atenção,
Bernardo

-- Mensagem original --

>Bernardo,
>
>  Boa tarde,
>
>> 
>> Prove que R - Q (o conjunto dos números Irracionais) não pode ser escrito
>> como uma união enumerável de conjuntos fechados.
>> 
> 
>  Se entendi o seu problema, ele pede para provar que, com a topologia
>usual de R, nao existem subconjuntos fechados de R, F_1, F_2, ..., F_n,
>..., tais que a reuniao dos F_n seja R-Q, certo?
>
>  Se for isso, suponha por abusrdo, que isso e' falso e seja, para cada
n,
>O_n = R - F_n.
>
>  O_n e' aberto, qualquer que seja n, e e' facil ver que a interseccao
dos
>O_n e' Q.
>
>  Como Q e' denso em R, e' claro que cada O_n e' denso em R.
>
>  Entao, como R e' um espaco de Baire (por ser completo), segue-se do
>teorema de Baire que a interseccao dos O_n e' um espaco de Baire. 
>
>  Mas isto e' um absurdo, pois Q e' enumeravel, e portanto e' a reuniao
>enumeravel de fechados sem interior nao podendo assim ser de Baire.
>
>Manuel Garcia
>
>
>=
>Instruções para entrar na lista, sair da lista e usar a lista em
>http://www.mat.puc-rio.br/~nicolau/olimp/obm-l.html
>=
>



--
Use o melhor sistema de busca da Internet
Radar UOL - http://www.radaruol.com.br



=
Instruções para entrar na lista, sair da lista e usar a lista em
http://www.mat.puc-rio.br/~nicolau/olimp/obm-l.html
=


[obm-l] Referencia Bibliografica (era: Combinatoria (In off))

2003-07-15 Por tôpico Manuel Valentim Pera
Paulo,

  Boa tarde,

On Tue, 15 Jul 2003, Paulo Santa Rita wrote:

> 
> Voce nao gostaria de apresentar aqui uma construcao dos reais, via cortes  
> ou sequencias de Cauchy, por exemplo, e desta construcao derivar o TEOREMA 
> DO SUPREMO ?
> 

 Nao teria sentido fazer isso aqui, mas recomendo a leitura dos excelentes
textos abaixo em que isso esta' bem feito, muito mais bem feito do que
qualquer coisa que eu pudesse esbocar aqui (eles podem ser encontrados em
bibliotecas de faculdades de Matematica):

  (A) Para construcoes via cortes de Dedekind: 

  Rudin, W.- Priciples of Real Analysis-3rd edition, McGraw Hill
International Editions (da segunda edicao deste livro ha', infelizmente,
uma traducao, mas essa edicao nao tras a cosntrucao de R). Veja o apendice
ao capitulo 1.

  Spivak, M.- Calculus (vol II) - Editorial Reverte, veja o apendice 1 
deste texto.

  (B) Para construcoes atraves de sequencias de Cauchy (particularmente
gosto mais deste enfoque):

  Esta' feita na seccao 5 do livro

  Hewitt, E. & Stromberg, K.- Real and Abstract Analysis - Springer.
  
Um ultimo comentario, historico, a construcao por cortes e' devida a
Dedekind (foi publicada pela promeira vez em 1872) e por sequencias de
Cauchy e' devida a Cantor, sendo originariamente publicada tambem em 1872.

Manuel Garcia

=
Instruções para entrar na lista, sair da lista e usar a lista em
http://www.mat.puc-rio.br/~nicolau/olimp/obm-l.html
=


Re: [obm-l] Análise Real

2003-07-15 Por tôpico Manuel Valentim Pera
Bernardo,

  Boa tarde,

> 
> Prove que R - Q (o conjunto dos números Irracionais) não pode ser escrito
> como uma união enumerável de conjuntos fechados.
> 
 
  Se entendi o seu problema, ele pede para provar que, com a topologia
usual de R, nao existem subconjuntos fechados de R, F_1, F_2, ..., F_n,
..., tais que a reuniao dos F_n seja R-Q, certo?

  Se for isso, suponha por abusrdo, que isso e' falso e seja, para cada n,
O_n = R - F_n.

  O_n e' aberto, qualquer que seja n, e e' facil ver que a interseccao dos
O_n e' Q.

  Como Q e' denso em R, e' claro que cada O_n e' denso em R.

  Entao, como R e' um espaco de Baire (por ser completo), segue-se do
teorema de Baire que a interseccao dos O_n e' um espaco de Baire. 

  Mas isto e' um absurdo, pois Q e' enumeravel, e portanto e' a reuniao
enumeravel de fechados sem interior nao podendo assim ser de Baire.

Manuel Garcia


=
Instruções para entrar na lista, sair da lista e usar a lista em
http://www.mat.puc-rio.br/~nicolau/olimp/obm-l.html
=


[obm-l] Análise Real

2003-07-15 Por tôpico bmat
Aqui vai um problema de Análise Real que está dando trabalho:

Prove que R - Q (o conjunto dos números Irracionais) não pode ser escrito
como uma união enumerável de conjuntos fechados.

Obrigado,
Bernardo



--
Use o melhor sistema de busca da Internet
Radar UOL - http://www.radaruol.com.br



=
Instruções para entrar na lista, sair da lista e usar a lista em
http://www.mat.puc-rio.br/~nicolau/olimp/obm-l.html
=


[obm-l] Re: [obm-l] Seu crêisson!!! O_Erro_Comum...

2003-07-15 Por tôpico J.Paulo roxer ´til the end



Vcs acham que esse é um posicionamento sério de 
quem gosta de matemática e se preocupa com a educação?
 
Murilo,não entrei aqui pra ler coisas inúteis 
nem estou disposto a perder meu tempo lendo piadinhas 
ridículas.
Vá engrandecer seu intelecto e dar respostas 
mais sábias quanto às minhas indagações.
 
João Paulo

  - Original Message - 
  From: 
  MuriloRFL 
  
  To: OBM-Lista 
  Sent: Tuesday, July 15, 2003 10:26 
  AM
  Subject: [obm-l] Seu crêisson!!! 
  O_Erro_Comum...
  
  Seu sinho JP,
   
   Nois, da 
  faucudadi seu Crêisson R$ ®, convidiamos vc pra nossia faucudadi 
  Nois temo aula especialica pra vc... Pra alunio reclamonico e griladiu!!! Temo 
  terapia de chókio A faucudade só necessitia di sua 
  assinaturia Prova nois num faiz pq nois num sabe mesmiu! nois 
  queremio é seu dinheirio E nem precisia escrivinha bestieiria!!! 
  Vc fiquia bem longi du lapisu.
   
          
  Gratilissimio,
   
     Faucudadi 
  seu Crêisson R$ ® 
   
  Agoria cum brilhiu nus olhius! 
  $-;-$
    
  X
   
   
  - Original Message - 
  From: J.Paulo 
  roxer ´til the end 
  To: [EMAIL PROTECTED] 
  Sent: Tuesday, July 15, 2003 1:55 AM
  Subject: [obm-l] Re: [obm-l] Re: [obm-l] 
  Re:_[obm-l]_O_Erro_Comum_da_Generalização_(_Ou_induções_equivocadas_)_
  
  Vc não se considera prepotente quando escreve coisas como "não admito que 
  NINGUÉM, especialmente um ignorante em Matemática " e "exijo que o sr 
  paulo recolha-se a sua insignificância "???
  Sou um inútil completo,não sei de nada,nunca 
  estudei,não estudo nem sirvo pra nada,correto?
  Certo,vc sabe de tudo,entende tudo que diz respeito à matemática,é o ser 
  supremo que merece a atenção de todos e merece ser muito respeitado.
   
  João Paulo



Email.it, the professional e-mail, gratis per te: clicca qui 

Sponsor:
Iscriviti Gratis al primo corso on-line di Web Marketing in Italia!  Collegati subito a: http://www.internet-marketing.it/?EMAILk0403a
Clicca qui 

[obm-l] Redução ao 1º quadrante

2003-07-15 Por tôpico Nelson alotiab

Olá a todos. Pra variar, gostaria de uma ajuda numa questão bem simples. Estarei sinceramente grato por qualquer ajuda.
 
[senx + cos(pi/2 - x)][cotg(x - pi) - cotg(2pi - x)]
 
Consegui responder essa questão:
cos(pi/2 - x) = senx
cotg(x - pi) = cotgx
cotg(2pi - x) = - cotgx
Substituindo:

[senx + senx)][cotgx - ( -cotgx)] = (2senx)(2cotgx) = (4senx)(cosx/senx) = 4cosx
Ok, mas, durante alguns dos meus "vôos", eu tinha pensado:
1º) cotg(x - pi), temos: x > pi;
2º) cotg(2pi - x), temos: x < 2pi; 
3º) 1º) inter 2º) = pi < x < 2pi 
e, agora, em cos(pi/2 - x), considerando 3º), temos:
4º) cos(pi/2 - x) = cos[(pi/2 - x) < 0]
Poderiamos também fazer outras inferências, mas, partindo dessa, eu só queria ratificar uma coisa: É Devido a simetria no ciclo trigonométrico que torna a resolução "gabarito" válida, e faz o meu último raciocínio mais um "grande vôo"?
 
Obrigado pela atenção,
NelsonYahoo! Mail 
Mais espaço, mais segurança e gratuito: caixa postal de 6MB, antivírus, proteção contra spam.

[obm-l] Redução ao 1º quadrante

2003-07-15 Por tôpico Nelson alotiab
Olá a todos. Pra variar, gostaria de uma ajuda numa questão bem simples. Estarei sinceramente grato por qualquer ajuda.
 
[senx + cos(pi/2 - x)][cotg(x - pi) - cotg(2pi - x)]
 
Consegui responder essa questão:
cos(pi/2 - x) = senx
cotg(x - pi) = cotgx
cotg(2pi - x) = - cotgx
Substituindo:

[senx + senx)][cotgx - ( -cotgx)] = (2senx)(2cotgx) = (4senx)(cosx/senx) = 4cosx
Ok, mas, durante alguns dos meus "vôos", eu tinha pensado:
1º) cotg(x - pi), temos: x > pi;
2º) cotg(2pi - x), temos: x < 2pi; 
3º) 1º) inter 2º) = pi < x < 2pi 
e, agora, em cos(pi/2 - x), considerando 3º), temos:
4º) cos(pi/2 - x) = cos[(pi/2 - x) < 0]
Poderiamos também fazer outras inferências, mas, partindo dessa, eu só queria ratificar uma coisa: É Devido a simetria no ciclo trigonométrico que torna a resolução "gabarito" válida, e faz o meu último raciocínio mais um "grande vôo"?
 
Obrigado pela atenção,
Nelson
 
 Yahoo! Mail 
Mais espaço, mais segurança e gratuito: caixa postal de 6MB, antivírus, proteção contra spam.

[obm-l] E-mail do Tengan sobre o IMO 6

2003-07-15 Por tôpico edmilson motta
Ei pessoal,

voces notaram que o problema 6 da prova e' uma
versao simplificada de um problema que eu e o Ed
mandamos em uma das listas de treinamento do ano
passado?  O problema da lista era algo assim:

Sejam a,r>1 e p um primo.  Prove que existe um
primo q tal que (a mod q) tem ordem p^r.

Este e' o famoso lema de van der Waerden, que e'
utilizado na prova do teorema de reciprocidade
geral de Artin (mais detalhes, veja por exemplo
Lang, Algebraic Number Theory, pag. 200).

A minha solucao e' curta demais pra um problema 6 da
IMO,
entao gostaria de pedir que voces checassem a 
solucao.  Para nao irritar aqueles que ainda
nao pensaram no problema, vou deixar um espaco
em branco:















































mais em baixo...




































































mais um pouco...















































ta' chegando...
































Agora sim, vamos ao problema.  Em primeiro lugar,
olhando para uma raiz
primitiva 
de q, e' facil reduzir o problema a
provar que existe um primo q tal que
p mod q nao e' uma p-esima potencia, i.e.,
p^{(q-1)/p} mod q nao e' 1 mod q.

Considere

N = (p^p-1)/(p-1) = p^(p-1) + ... + p + 1

Se q e' um primo que divide N e p-1, entao de
N=p mod q, segue q=p, absurdo.  Entao para
todo primo q que divide N, p mod q tem ordem
exatamente p.  O problema acaba se p^2 nao
divide q-1, mas se todos os primos que dividem
N sao = 1 mod p^2, entao N = 1 mod p^2, o
que e' um absurdo.

Agora vejam: se no lema de van der Waerden
a=p, r=1, este e' exatamente o problema da IMO,
com algumas pequenas modificacoes!  A solucao
do problema da lista e' igualzinho `a
demonstracao acima.  Eu lembro que o Alex e o
Issao fizeram este problema, e acho que mais
alunos tambem acertaram.  Espero que os
pokemons tenham se lembrado do problema
durante a prova!

Estamos melhorando: um problema na IMO e uma
previsao acertada!  Alguem arrisca os proximos
numeros da loto?

Ate'
ET



__
Do you Yahoo!?
SBC Yahoo! DSL - Now only $29.95 per month!
http://sbc.yahoo.com
=
Instruções para entrar na lista, sair da lista e usar a lista em
http://www.mat.puc-rio.br/~nicolau/olimp/obm-l.html
=


[obm-l] Progressões: EXTREMAMENTE.......

2003-07-15 Por tôpico MuriloRFL
segundo o maple,
S(15092689) = 100.000
S(1.509268*10^43)
> > Eh claro que S(12 000) nao eh igual a 10 exatamente, Alexandre.
>
> Segundo o Maple, S(12000) = 9,969919260.
>
> Abraço,
> Henrique.
>
> =
> Instruções para entrar na lista, sair da lista e usar a lista em
> http://www.mat.puc-rio.br/~nicolau/olimp/obm-l.html
> =
>

=
Instruções para entrar na lista, sair da lista e usar a lista em
http://www.mat.puc-rio.br/~nicolau/olimp/obm-l.html
=


[obm-l] dificuldade

2003-07-15 Por tôpico Fabrício




Prove que:se a,b e c são lados de um triângulo e 
satisfaz a sentença a^2+b^2+c^2=9r^2, onde r é o 
raio da circunferência circunscrita, então esse triângulo é 
eqüilátero.


[obm-l] Re: [obm-l] RE: [obm-l] RE: [obm-l] Progressões: EXTREMAMENTE.......

2003-07-15 Por tôpico Henrique Patrício Sant'Anna Branco
> Eh claro que S(12 000) nao eh igual a 10 exatamente, Alexandre.

Segundo o Maple, S(12000) = 9,969919260.

Abraço,
Henrique.

=
Instruções para entrar na lista, sair da lista e usar a lista em
http://www.mat.puc-rio.br/~nicolau/olimp/obm-l.html
=


[obm-l] Problemas IMO - Questao 4

2003-07-15 Por tôpico latino
So um pequeno detalhe... nao precisei usar o fato de ABCD ser incritivel
(pelo menos nao explicitamente). Alguem poderia comentar isso?

#
# MSc. Edson Ricardo de A. Silva#
# Computer Graphics Group (CRAB)#
# Federal University of Ceara (UFC) #
#

> achei legal essa sua solucao por complexos. Uma outra solucao
> trivial (e acho que a de 99% dos participantes) seria a seguinte:
>
> quad. APDR inscritivel  =>  PR = AD.sen( quad. CQRD inscritivel  =>  RQ = DC.sen(
> PR = RQ  =>  AD/DC = sen(
> Sendo S e T os pontos de interseccao das bissetrizes internas dos
> angulos 
> AS/SC  =  AB/BC  =  AD/DC  =  AT/TC   Logo, S = T
>   (1)   (2)   (3)
>
> (1) e (3) - teorema da bissetriz interna
> (2) - por (*)
>
> abracos,
=
Instruções para entrar na lista, sair da lista e usar a lista em
http://www.mat.puc-rio.br/~nicolau/olimp/obm-l.html
=


Re: [obm-l] IMO - P1

2003-07-15 Por tôpico Marcio Afonso A. Cohen
Acho que consegui fazer  o 1o. Confiram ai e vejam se tem algum furo. O
2o eu realmente nao estou conseguindo.. Estou com alguma esperanca de fazer
o 5.. (o 3 eu tentei tmb, mas minhas contas estao muito grandes). Mandem
seus comentarios sobre a prova!
P1:
Note que (Ai inter Aj) != vazio sse existirem m,n tais que a_m + t_i =
a_n + t_j , i.e, a_m - a_n = t_j - t_i.
Vamos construir os t's indutivamente garantindo que isso nao acontece.
Existem binomial (101,2) = 5050 diferencas possiveis no conjunto A. Chame de
D={D1,D2,...D5050} o conjunto dessas diferencas (claro que algumas delas
podem ser iguais, mas temos |D| <= 5050).
1. Escolha um t1 qualquer de S.
2. Agora quero garantir que t2-t1 e t1-t2 nao estao em D. Para isso, basta
escolher um elemento de S que nao esteja em
X1 = {t1+D1,t1+D2,...,t1+D5050}U{t1-D1, t1-D2,...,t1-D5050}. (pq se t2-t1
esta em D, entao t2=t1+Dk para algum k).
Isso eh facil pq |X1|<=2.5050 < |S|.

3. Agora vou escolher t3 em S garantindo que t3-t1, t1-t3, t3-t2, t2-t3 nao
estao em D.
Para isso, t3 nao pode estar em X1 e tmb nao pode estar em
X2 = {t2+D1,t2+D2,...,t2+D5050}U{t2-D1, t2-D2,...,t2-D5050}.
Isso eh facil, pq |X1 U X2| <= 4.5050 < |S|

Em geral, depois de escolhidos t1,t2,...,t_k-1, vou escolher t_k em S de
modo que ele nao esteja em nenhum dos conjuntos X1,X2,...,X_(k-1).
Para k<=100, isso eh sempre possivel, pq |X1 U X2 U ... U X_(k-1)| <=
2*(k-1)*5050 <= 2*99*5050 = 00 < 10^6 = |S|.
(obs: X_s = {ts + D}U{ts-D}, na notacao usuao de x+A onde x eh um elemento e
A um conjunto).


Pronto. Foram escolhidos 100 t's tal que nao existe uma quadrupla (m,n,i,j)
tq a_m - a_n = t_j - t_i. (pois t_j - t_i esta sempre fora de D), e portanto
nunca se tem a_m + t_i = a_n + t_j, ou seja, as intersecoes sao todas vazias
de fato.

Abracos.



- Original Message -
From: <[EMAIL PROTECTED]>
To: <[EMAIL PROTECTED]>
Cc: <[EMAIL PROTECTED]>; <[EMAIL PROTECTED]>
Sent: Monday, July 14, 2003 3:38 PM
Subject: [obm-l] Problemas da IMO


>
>
> Prova da IMO retirada do Site http://www.mathlinks.go.ro/
>
> O Problema 1 é nois que mandou...
>
>
> First Day - 44th IMO 2003 Japan
>
> 1. Let A be a 101-element subset of the set S={1,2,3,...,100}. Prove
that
> there exist numbers t_1, t_2, ..., t_{100} in S such that the sets
>
> Aj = { x + tj | x is in A } for each j = 1, 2, ..., 100
>
> are pairwise disjoint.
>
>
> 2. Find all pairs of positive integers (a,b) such that the number
>
> a^2 / ( 2ab^2-b^3+1) is also a positive integer.
>
> 3. Given is a convex hexagon with the property that the segment connecting
the
> middle points of each pair of opposite sides in the hexagon is  sqrt(3) /
2
> times the sum of those sides' sum.
>
> Prove that the hexagon has all its angles equal to 120.
>
>
> Second Day - 44th IMO 2003 Japan
>
> 4. Given is a cyclic quadrilateral ABCD and let P, Q, R be feet of the
> altitudes from D to AB, BC and CA respectively. Prove that if PR = RQ then
the
> interior angle bisectors of the angles < ABC and < ADC are concurrent on
AC.
>
> 5. Let x1 <= x2 <= ... <= xn be real numbers, n>2.
>
> a) Prove the following inequality:
>
> (sum  ni,j=1 | xi - xj | ) 2 <= 2/3 ( n^2 - 1 )sum ni,j=1 ( xi - xj)^2
>
> b) Prove that the equality in the inequality above is obtained if and only
if
> the sequence (xk) is an arithemetical progression.
>
> 6. Prove that for each given prime p there exists a prime q such that
n^p - p
> is not divisible by q for each positive integer n.
>
>
>
> -
> This mail sent through IMP: http://horde.org/imp/
> =
> Instruções para entrar na lista, sair da lista e usar a lista em
> http://www.mat.puc-rio.br/~nicolau/olimp/obm-l.html
> =
>

=
Instruções para entrar na lista, sair da lista e usar a lista em
http://www.mat.puc-rio.br/~nicolau/olimp/obm-l.html
=


Re: [obm-l] Problemas da IMO

2003-07-15 Por tôpico latino
Marcio,
achei legal essa sua solucao por complexos. Uma outra solucao
trivial (e acho que a de 99% dos participantes) seria a seguinte:

quad. APDR inscritivel  =>  PR = AD.sen(  RQ = DC.sen(  AD/DC = sen( Eu sei que ninguem gosta muito disso, mas esse problema 4 (que eu ateh
> imagino que nao seja dificil por plana) eh bem simples na conta bruta.. Eh
> impressionante como complexos ajudam nos problemas de geometria da imo..
> aquele artigo da eureka 6 eh realmente muito util!
>
> Coloque o circuncentro na origem, e represente os vertices pelos
> complexos a,b,c,d, todos de modulo 1u.m.
> Reta ab: z+abz' = a+b
> Reta perpendicular a ab passando por d: z-abz'=d-abd'
> Logo, o ponto P eh 2p = [a+b+d-ab/d]
> Portanto, 2q = [a+c+d-ac/d] e 2r = [b+c+d-bc/d].
> Como p,q,r sao colineares (reta de simpson), e |p-q| = |q-r|:
> p-q = q-r, ou seja: b-c + ac/d - ab/d = a-b +bc/d-ac/d
> Arrumando: (b-c) - (a/d)(b-c) = (a-b) - (c/d)(a-b) sse (b-c)(d-a)=(a-b)(d-c)
> Tirando modulo, isso significa que BC*AD = AB*DC. E isso fecha o problema.
> De fato, sendo I o peh da bissetriz de ABC em AC, entao, AI/IC = AB/BC e vc
> quer provar que I eh peh da bissetriz de ADC, i.e, que AI/IC=AD/DC (teorema
> da bissetriz interna, ida e volta). Portanto, eh suficiente provar que AB*DC
> = AD*BC.
>
> Vou pensar nos outros agora, esse foi o que eu achei que seria mais
> facil.. (ja pensei no 2 e no 1 um pouco tmb..)
>
>
> - Original Message -
> From: <[EMAIL PROTECTED]>
> To: <[EMAIL PROTECTED]>
> Cc: <[EMAIL PROTECTED]>; <[EMAIL PROTECTED]>
> Sent: Monday, July 14, 2003 3:38 PM
> Subject: [obm-l] Problemas da IMO
>
>
> >
> >
> > Prova da IMO retirada do Site http://www.mathlinks.go.ro/
> >
> > O Problema 1 é nois que mandou...
> >
> >
> > First Day - 44th IMO 2003 Japan
> >
> > 1. Let A be a 101-element subset of the set S={1,2,3,...,100}. Prove
> that
> > there exist numbers t_1, t_2, ..., t_{100} in S such that the sets
> >
> > Aj = { x + tj | x is in A } for each j = 1, 2, ..., 100
> >
> > are pairwise disjoint.
> >
> >
> > 2. Find all pairs of positive integers (a,b) such that the number
> >
> > a^2 / ( 2ab^2-b^3+1) is also a positive integer.
> >
> > 3. Given is a convex hexagon with the property that the segment connecting
> the
> > middle points of each pair of opposite sides in the hexagon is  sqrt(3) /
> 2
> > times the sum of those sides' sum.
> >
> > Prove that the hexagon has all its angles equal to 120.
> >
> >
> > Second Day - 44th IMO 2003 Japan
> >
> > 4. Given is a cyclic quadrilateral ABCD and let P, Q, R be feet of the
> > altitudes from D to AB, BC and CA respectively. Prove that if PR = RQ then
> the
> > interior angle bisectors of the angles < ABC and < ADC are concurrent on
> AC.
> >
> > 5. Let x1 <= x2 <= ... <= xn be real numbers, n>2.
> >
> > a) Prove the following inequality:
> >
> > (sum  ni,j=1 | xi - xj | ) 2 <= 2/3 ( n^2 - 1 )sum ni,j=1 ( xi - xj)^2
> >
> > b) Prove that the equality in the inequality above is obtained if and only
> if
> > the sequence (xk) is an arithemetical progression.
> >
> > 6. Prove that for each given prime p there exists a prime q such that
> n^p - p
> > is not divisible by q for each positive integer n.
> >
> >
>
> =
> Instruções para entrar na lista, sair da lista e usar a lista em
> http://www.mat.puc-rio.br/~nicolau/olimp/obm-l.html
> =
>
=
Instruções para entrar na lista, sair da lista e usar a lista em
http://www.mat.puc-rio.br/~nicolau/olimp/obm-l.html
=


Re: [obm-l] Combinatoria (In off)

2003-07-15 Por tôpico Paulo Santa Rita
Ola Manuel e demais
colegas desta lista ... OBM-L,
1) A sua mensagem, nao obstante nao tratar de algum problema especifico, e 
muito boa. Ela realmente enriquece a Lista e e o tipo de OFF que espera-se 
que ocorra.

2) A Matematica continua e continuara sendo o tema desta Lista. Ainda que 
algumas pessoas mal educadas e mal formadas estejam - acredito firmemente - 
deliberadamente atentando contra estes objetivos originais, ela se mantera 
com a alta qualidade que nos, Professores, Pesquisadores e estudantes 
serios, almejamos e que lutamos para que tenha,

3) E verdade que o AXIOMA DO SUPREMO pode surgir como uma propriedade em 
alguma construcao particular dos numeros reais ou pode ser adotado como mais 
um axioma em outras construcoes. No livro de Analise I, Projeto Euclides. O 
Prof Elon adota este ultima postura, mas explicitamente cita outros obras 
nas quais os reais sao construidos e diz que uma tal construcao e um 
processo instrutivo. Em sintese, nao existe nenhuma razao mais forte para se 
adotar uma vertente ou outra, alem de crencas subjetivas e individuais.

4) A imensa maioria das teorias matematicas surgem de forma altamente 
intuitivas, pouco formais, somente rrecebendo um tratamento axiomatico 
posteriormente. Foi assim com o Calculo, com a Topologia, com a teoria dos 
grupos e com muitas ( talvez todas ! ) outras teorias. A intuicao vai na 
frente, descobre e orienta a pesquisa;  a formalizacao ou axiomatizacao vem 
depois e fundamenta com rigor as conquistas ja feitas. A primeira e a 
faculdade da descoberta, atributo do genio; a segunda, ferramente de prova e 
de resolucao de problemas, obra do talento.

Voce nao gostaria de apresentar aqui uma construcao dos reais, via cortes  
ou sequencias de Cauchy, por exemplo, e desta construcao derivar o TEOREMA 
DO SUPREMO ?

Um Abraco
Paulo Santa Rita
3,1154,150703

From: Manuel Valentim Pera <[EMAIL PROTECTED]>
Reply-To: [EMAIL PROTECTED]
To: [EMAIL PROTECTED]
Subject: Re: [obm-l] Combinatoria (In off)
Date: Mon, 14 Jul 2003 19:11:05 -0300 (EST)
Boa noite,

  Sobre o trecho:

>
> O segundo caso (mais geral) que você colocou, realmente merece uma
> demonstração, eu acho.
> Mas na minha cabeça, esse Princípio de Dirichlet seria uma coisa tão
> intuitiva que não precisaria de provas.
> Aí eu me embolo... Quando uma proposição precisa ser provada e quando se
> admite que ela é intuitiva o suficiente para ser aceita sem
demonstração?
>
  No sentido que as palavras tem em matematica (e matematica era, ate'
algum tempo atras, o assunto desta lista) sua duvida nao tem uma resposta
"absoluta", exceto a trivial, DEPENDE do que foi admitido como axioma no
contexto de seu estudo, isto inclui, entre outras coisas mais mundanas,
quais os axiomas de teoria dos conjuntos que voce esta' admitindo. Tudo o
que nao for axioma precisa ser demonstrado.
  Eu nunca vi o principio de Dirichlet (ou pigeonhole) ser colocado como
axioma, entao precisa de uma demonstracao (se voce estiver admitindo os
postulados de Peanno para o conjunto dos naturais e ZF, isso sai
trivialmente, mas e' a demonstracao que e' trivial, nao a afirmacao. Alias
essa afirmacao tao "trivial" caracteriza, em muitos contextos, conjuntos
finitos), mas esta frase diz apenas isso: "eu", na minha limitadissima
experiencia, nunca vi...
  Poucas afirmacoes sao tao "evidentes" (maldita palavra) como a do
"Teorema da Curva de Jordan", se alguem conhecer alguma demonstracao
"trivial" dela, por favor, mostre-ma!
  Apenas um adendo, COM O AVISO DE IN-OFF EM MAIUSCULAS.

  Axioma, em matematica, nada tem a ver com "intuitivo", ou "evidente". os
axiomas das geometrias nao-euclideanas sao, do ponto de vista matematico
(outros pontos de vista deveriam ser assunto de bate papo em mesa de
botequim, coisas muito interessantes alias essas conversas, mas nao desta
lista), tao "intuitivas" quanto as euclideanas.
  Num exemplo concreto, falou-se nesta lista ha' nao muito tempo em
"axioma do supremo para o conjunto dos Reais", isso ser "um axioma" so'
faz sentido numa teoria em que o conjunto dos numeros Reais (R) e'
"apresentado" axiomaticamente. Se voce quiser <>, por exemplo a
partir dos numeros racionais, esse conjunto isso deixa de ser "axioma" e
passa a ser uma PROPRIEDADE e precisa ser demonstrada.
  Hoje em dia pode parecer estranho falar-se em "construcao" de R, pois
o "metodo axiomatico" e' a unica forma que i conjunto dos Reais e'
apresentado (como dizia N. Rodrigues, "toda unaminidade e' burra") e
construcoes de R sao temas desconhecidos dos dois primeiros anos de cursos
de graduacao em matematica, mas para pelo menos um grande Professor de
Matematica que eu conheco isso esta' longe de ser uma virtude do atual
"modelo de ensino"...
Desculpem o carater in-off do adendo, ele foge completamente dos objetivos
desta lista, mas em virtude de certas "perolas" recentes, que nem ao menos
vem com o aviso de in-off no "subject" penso que o professor Nicolau
perdoara' este deslize.
Manuel Garcia

===

Re: [obm-l] Revista Eureka. No. 16

2003-07-15 Por tôpico matemandreca
Caro, Nelly e amigos da lista, 
Gostaria de saber como faço para que o colégio onde trabalho passe a receber 
a Eureka e maiores informações sobre as olimpiadas. Há alunos com interesse 
em participar. 
Desde já grato, André. 



Em 19 Jun 2003, [EMAIL PROTECTED] escreveu: 

>Caros(as) amigos(as) da lista: 
> 
>Ja' estamos distribuindo as revistas Eureka No. 16 
>para todas as Coordenacoes Regionais, 
>Colegios Cadastrados na OBM e Assinantes. 
> 
>Abracos, Nelly. 
> 
>= 
>Instruções para entrar na lista, sair da lista e usar a lista em 
>http://www.mat.puc-rio.br/~nicolau/olimp/obm-l.html 
>= 
> 
>-- 

_
Voce quer um iGMail protegido contra vírus e spams? 
Clique aqui: http://www.igmailseguro.ig.com.br
Ofertas imperdíveis! Link: http://www.americanas.com.br/ig/

=
Instruções para entrar na lista, sair da lista e usar a lista em
http://www.mat.puc-rio.br/~nicolau/olimp/obm-l.html
=


[obm-l] Seu crêisson!!! O_Erro_Comum...

2003-07-15 Por tôpico MuriloRFL



Seu sinho JP,
 
 Nois, da 
faucudadi seu Crêisson R$ ®, convidiamos vc pra nossia faucudadi 
Nois temo aula especialica pra vc... Pra alunio reclamonico e griladiu!!! Temo 
terapia de chókio A faucudade só necessitia di sua assinaturia 
Prova nois num faiz pq nois num sabe mesmiu! nois queremio é seu 
dinheirio E nem precisia escrivinha bestieiria!!! Vc fiquia bem longi 
du lapisu.
 
        
Gratilissimio,
 
   Faucudadi 
seu Crêisson R$ ® 
 
Agoria cum brilhiu nus olhius! 
$-;-$
  
X
 
 
- Original Message - 
From: J.Paulo roxer 
´til the end 
To: [EMAIL PROTECTED] 
Sent: Tuesday, July 15, 2003 1:55 AM
Subject: [obm-l] Re: [obm-l] Re: [obm-l] 
Re:_[obm-l]_O_Erro_Comum_da_Generalização_(_Ou_induções_equivocadas_)_

Vc não se considera prepotente quando escreve coisas como "não admito que 
NINGUÉM, especialmente um ignorante em Matemática " e "exijo que o sr 
paulo recolha-se a sua insignificância "???
Sou um inútil completo,não sei de nada,nunca 
estudei,não estudo nem sirvo pra nada,correto?
Certo,vc sabe de tudo,entende tudo que diz respeito à matemática,é o ser 
supremo que merece a atenção de todos e merece ser muito respeitado.
 
João Paulo


Re: [obm-l] Problemas da IMO

2003-07-15 Por tôpico Marcio Afonso A. Cohen
Eu sei que ninguem gosta muito disso, mas esse problema 4 (que eu ateh
imagino que nao seja dificil por plana) eh bem simples na conta bruta.. Eh
impressionante como complexos ajudam nos problemas de geometria da imo..
aquele artigo da eureka 6 eh realmente muito util!

Coloque o circuncentro na origem, e represente os vertices pelos
complexos a,b,c,d, todos de modulo 1u.m.
Reta ab: z+abz' = a+b
Reta perpendicular a ab passando por d: z-abz'=d-abd'
Logo, o ponto P eh 2p = [a+b+d-ab/d]
Portanto, 2q = [a+c+d-ac/d] e 2r = [b+c+d-bc/d].
Como p,q,r sao colineares (reta de simpson), e |p-q| = |q-r|:
p-q = q-r, ou seja: b-c + ac/d - ab/d = a-b +bc/d-ac/d
Arrumando: (b-c) - (a/d)(b-c) = (a-b) - (c/d)(a-b) sse (b-c)(d-a)=(a-b)(d-c)
Tirando modulo, isso significa que BC*AD = AB*DC. E isso fecha o problema.
De fato, sendo I o peh da bissetriz de ABC em AC, entao, AI/IC = AB/BC e vc
quer provar que I eh peh da bissetriz de ADC, i.e, que AI/IC=AD/DC (teorema
da bissetriz interna, ida e volta). Portanto, eh suficiente provar que AB*DC
= AD*BC.

Vou pensar nos outros agora, esse foi o que eu achei que seria mais
facil.. (ja pensei no 2 e no 1 um pouco tmb..)


- Original Message -
From: <[EMAIL PROTECTED]>
To: <[EMAIL PROTECTED]>
Cc: <[EMAIL PROTECTED]>; <[EMAIL PROTECTED]>
Sent: Monday, July 14, 2003 3:38 PM
Subject: [obm-l] Problemas da IMO


>
>
> Prova da IMO retirada do Site http://www.mathlinks.go.ro/
>
> O Problema 1 é nois que mandou...
>
>
> First Day - 44th IMO 2003 Japan
>
> 1. Let A be a 101-element subset of the set S={1,2,3,...,100}. Prove
that
> there exist numbers t_1, t_2, ..., t_{100} in S such that the sets
>
> Aj = { x + tj | x is in A } for each j = 1, 2, ..., 100
>
> are pairwise disjoint.
>
>
> 2. Find all pairs of positive integers (a,b) such that the number
>
> a^2 / ( 2ab^2-b^3+1) is also a positive integer.
>
> 3. Given is a convex hexagon with the property that the segment connecting
the
> middle points of each pair of opposite sides in the hexagon is  sqrt(3) /
2
> times the sum of those sides' sum.
>
> Prove that the hexagon has all its angles equal to 120.
>
>
> Second Day - 44th IMO 2003 Japan
>
> 4. Given is a cyclic quadrilateral ABCD and let P, Q, R be feet of the
> altitudes from D to AB, BC and CA respectively. Prove that if PR = RQ then
the
> interior angle bisectors of the angles < ABC and < ADC are concurrent on
AC.
>
> 5. Let x1 <= x2 <= ... <= xn be real numbers, n>2.
>
> a) Prove the following inequality:
>
> (sum  ni,j=1 | xi - xj | ) 2 <= 2/3 ( n^2 - 1 )sum ni,j=1 ( xi - xj)^2
>
> b) Prove that the equality in the inequality above is obtained if and only
if
> the sequence (xk) is an arithemetical progression.
>
> 6. Prove that for each given prime p there exists a prime q such that
n^p - p
> is not divisible by q for each positive integer n.
>
>

=
Instruções para entrar na lista, sair da lista e usar a lista em
http://www.mat.puc-rio.br/~nicolau/olimp/obm-l.html
=